Search found 11 matches


Hi Fabio, In this case if -1 is the lowest, then -1 could be largest also, so then the remaining terms could be existing or not existing also.. What do you think about this? No, allizzwell! The question stem says that -1 belongs to the set, and that for EVERY t element that is there (and -1 is cert...

by jeetu_vishnoi

Tue Oct 26, 2010 3:20 am
Forum: Data Sufficiency
Topic: Set theory with number system
Replies: 9
Views: 1857

Hi Fabio, Thanks for the reply but i have one query regarding your answer that question says that "A set of numbers" so there may be infinite numbers since no limit is given of the set. It never says that -1 is the first or lowest number then how can we say that -3 will not be in the set i...

by jeetu_vishnoi

Mon Oct 25, 2010 2:55 pm
Forum: Data Sufficiency
Topic: Set theory with number system
Replies: 9
Views: 1857

mean of the given S = (0+2+4+5+8+11)/6 = 5
to obtain the median arrange the S in ascending order S = {0,2,4,5,8,11}
now median is (4+5)/2 = 4.5
hence the difference between mean and median = 5 - 4.5 = 0.5

by jeetu_vishnoi

Mon Oct 25, 2010 2:50 pm
Forum: Problem Solving
Topic: how much greater..........
Replies: 2
Views: 1202

2x-3b = 0
b = 2x/3
since it is given that b<2 hence 2x/3 <2 or
2x <6 or
x<3

by jeetu_vishnoi

Mon Oct 25, 2010 2:45 pm
Forum: Problem Solving
Topic: which of the follwing must be true.........
Replies: 4
Views: 1170

Since cone is inscribed in a hemisphere conciding with the base of hemisphere then the height of the cone must be touching the side of hemisphere hence the height will be equal to the radius of the hemisphere and ratio is 1:1.

by jeetu_vishnoi

Mon Oct 25, 2010 2:42 pm
Forum: Problem Solving
Topic: Right circular cone question?
Replies: 6
Views: 3198

if we factorise 72 then it is 2*2*2*3*3
it is given that n2 is divisible by 72 hence if we make the above facorisation as square number then we have to multiply it by 2:
(2*2)(2*2)(3*3)
now from here you can see it easily that the largest number which can divide n is 2*2*3 = 12

by jeetu_vishnoi

Mon Oct 25, 2010 2:31 pm
Forum: Problem Solving
Topic: BTG 300 unclear solution
Replies: 7
Views: 1516

In these type of question you can only select a given condition as true iff it is always true. If there is even a single condition in which it can be false then you can not choose this condition as sufficient. In the given question you can see that in the first condition, required number will always...

by jeetu_vishnoi

Mon Oct 25, 2010 2:25 pm
Forum: Problem Solving
Topic: GMATprep question
Replies: 5
Views: 1376

if bigger rim revolves y times per seconds then according to the given condition:

2*pi*28*x = 2*pi*35*y

y = .8x

now we need to give the total revolution per minute hence answer = 60*(.8x) = 48x

by jeetu_vishnoi

Mon Oct 25, 2010 2:18 pm
Forum: Problem Solving
Topic: Circular rim
Replies: 8
Views: 5322

Answer is A. first equation can be written as (x-3)(x-1)<0 which means that (x-3)<0 and (x-1)>0 it shows that x<3 and x>1 and it is given that x is integer hence x=2. first equation can be written as (x+3)(x+1)<0 which means that either {(x+3)<0 and (x+1)<0} or {(x+3)>0 and (x+1)>0} which clearly sh...

by jeetu_vishnoi

Mon Oct 25, 2010 2:11 pm
Forum: Data Sufficiency
Topic: Gmat practice question
Replies: 9
Views: 2086

E becoz it is not possible to give a definite answer by given both conditions

by jeetu_vishnoi

Mon Oct 25, 2010 2:02 pm
Forum: Data Sufficiency
Topic: computer and printers
Replies: 7
Views: 4075

Set theory with number system

A set of numbers has the property that for any number t in the set, t+2 is in the set. If -1 is in the set, which one of the following must also be in the set?
i. -3
ii. 1
iii. 5

(A) i only
(B) ii only
(C) i and ii only
(D) ii and iii only
(E) i, ii and iii

by jeetu_vishnoi

Mon Oct 25, 2010 1:58 pm
Forum: Data Sufficiency
Topic: Set theory with number system
Replies: 9
Views: 1857